Безумные дельты Дирака

Я не ожидаю какой-либо строгости в следующем и ответах... так как мы имеем дело с дельтами Дирака в контексте КТП.

Рассмотрим интеграл

д 4 д   Θ ( д 0 ) Θ ( п 3 , 0 + д 0 )   дельта ( ( п 3 + д ) 2 ) дельта ( ( д 2 ) ) 1 ( д + п 2 ) 2 м 2 + я ϵ

The п 'песок д все 4-векторы Минковского и   п 3 "=" п 1 + п 2 . Для простоты можно работать в п 3 CM-Frame так, чтобы   п 3 "=" ( М , 0 ) .

У меня проблемы с дельтами (поскольку они связаны), я думал о д 0 сначала интеграция , как бы вы, ребята, приступили к выполнению д 0 -интеграция?. Я использую формулу состава для Dirac delta Dirac Delta Comp. Википедия , но когда я заменяю д 0 в аргументе другой дельты я получаю что-то вроде дельта ( М ) где М это масса п 3 . Эта дельта не имеет для меня смысла, поскольку мы даже не интегрируем М ? Кто-нибудь знает, как сделать этот интеграл с самого начала? Любой намек приветствуется, спасибо.

Здесь уже есть некоторые упрощения. В рамках CM вы предлагаете, включая оба Θ [ ( п 3 ) 0 + д 0 ] "=" Θ [ М + д 0 ] и Θ ( д 0 ) является избыточным.
Хорошо, но если мы забудем о тетах, как бы вы продолжили вычислять интегралы q_0? Дельты связаны, и я действительно не знаю, что делать.
Вторая дельта-функция говорит вам, что д 0 "=" ± | д | , а из-за Хевисайда возможен только знак +. Следует подумать о цепном правиле: дельта ( ф ( Икс ) ) "=" при разработке этого.
Да, именно так я и делаю... и затем, когда я подставляю это положительное q_0 в аргумент второй дельты, я получаю странный результат delta(M). Может быть, мне не следует просто подключать это. Жаль, что эти операции действительно волнистые, и я, к сожалению, никогда не видел их должным образом определенными. Например, в главе 10 Средницкого (если я правильно помню) он использует дельта ( Икс ) 2 "=" дельта ( Икс ) дельта ( 0 ) и т. д...
Средненицкий экв. 11.13, например: ' web.physics.ucsb.edu/~mark/ms-qft-DRAFT.pdf
Придумайте более подходящее название.

Ответы (1)

Я получаю ноль.

С использованием п 3 , 0 "=" М и предполагая М > 0 в Θ ( п 3 , 0 + д 0 ) фактор является избыточным, и у вас есть:

д 4 д   Θ ( д 0 )   дельта ( ( п 3 + д ) 2 ) дельта ( д 2 ) 1 ( д + п 2 ) 2 м 2 + я ϵ

Сейчас

дельта ( ( п 3 + д ) 2 ) дельта ( д 2 ) "=" дельта ( ( М + д 0 ) 2 д 2 ) дельта ( д 0 2 д 2 ) "=" дельта ( ( М + | д | ) 2 д 2 ) дельта ( д 0 2 д 2 ) "=" дельта ( М 2 + 2 М | д | ) дельта ( д 0 2 д 2 ) "=" 1 2 М дельта ( | д | + М / 2 ) дельта ( д 0 2 д 2 )

Первая строка просто вставляется. Во второй строке используется д 0 "=" + | д | благодаря второй дельта-функции и тета-функции. Третья строка просто расширяет и упрощает первую дельту. Последняя строка — это идентификатор правила цепочки для дельта-функции.

Сейчас дельта ( | д | + М / 2 ) имеет поддержку только отрицательных | д | что явная ерунда, поэтому интеграл равен нулю. Если у тебя есть п 3 д в исходном интеграле вместо п 3 + д вы получите ненулевой ответ.


В общем, вы можете использовать дельта ( Икс а ) дельта ( Икс б ) "=" дельта ( Икс а ) дельта ( а б ) так как первая дельта убивает интеграл везде, кроме Икс "=" а . Если у вас есть а "=" б Вы получаете дельта ( Икс а ) 2 "=" дельта ( Икс а ) дельта ( 0 ) и дельта ( 0 ) должен быть определен каким-то предписанием, например, положить систему в коробку.

Моя ошибка заключалась в том, что я НЕ заменил q_0 сразу, а вместо этого заменил ПОСЛЕ того, как я написал вторую дельту в форме композиции ... это неправильно, этого делать нельзя. Нужно немедленно заменить, как это сделал Майкл Браун.